Physics (10th Edition)

Published by Wiley
ISBN 10: 1118486897
ISBN 13: 978-1-11848-689-4

Chapter 7 - Impulse and Momentum - Problems - Page 196: 66

Answer

$v_{f1}=0.65m/s$ and $\theta=10^o$

Work Step by Step

1) On the $y$ axis: - Ball 1 has mass $m_1=0.15kg$, initial vertical velocity $v_{01}=-v_{01}\cos50=-0.9\cos50=-0.58m/s$ and final vertical velocity $v_{f1}=+v_{f1}\sin\theta (m/s)$. - Ball 2 has mass $m_2=0.26kg$, initial vertical velocity $v_{02}=-v_{02}\cos50=0$ and final vertical velocity $v_{f2}=-0.7\sin35=-0.4m/s$. Using conservation of momentum, we have $$m_1v_{01}+m_2v_{02}=m_1v_{f1}+m_2v_{f2}$$ $$-0.087+0=0.15v_{f1}\sin\theta-0.104$$ $$0.15v_{f1}\sin\theta=0.017$$ $$v_{f1}\sin\theta=0.113 (1)$$ 2) On the $x$ axis: - Ball 1 has initial horizontal velocity $+0.9\sin50=+0.69m/s$ and final horizontal velocity $+v_{f1}\cos\theta (m/s)$. - Ball 2 has initial horizontal velocity $v_{02}=+0.54m/s$ and final horizontal velocity $+0.7\cos35=+0.57m/s$. Using conservation of momentum, we have $$0.15(+0.69)+0.26(+0.54)=0.15(+v_{f_1}\cos\theta)+0.26(+0.57)$$ $$0.104+0.140=0.15v_{f_1}\cos\theta+0.148$$ $$0.15v_{f_1}\cos\theta=0.096$$ $$v_{f_1}\cos\theta=0.64 (2)$$ Divide (2) over (1): $$\frac{v_{f_1}\sin\theta}{v_{f_1}\cos\theta}=\tan\theta=\frac{0.113}{0.64}=0.177$$ $$\theta=10.037^o$$ So $$v_{f1}=\frac{0.113}{\sin10.037}=0.65m/s$$
Update this answer!

You can help us out by revising, improving and updating this answer.

Update this answer

After you claim an answer you’ll have 24 hours to send in a draft. An editor will review the submission and either publish your submission or provide feedback.